K
Khách

Hãy nhập câu hỏi của bạn vào đây, nếu là tài khoản VIP, bạn sẽ được ưu tiên trả lời.

29 tháng 2 2020

BĐT <=> \(\sqrt{\frac{x+yz}{xyz}}+\sqrt{\frac{y+xz}{xyz}}+\sqrt{\frac{z+xy}{xyz}}\ge1+\sqrt{\frac{1}{xy}}+\sqrt{\frac{1}{yz}}+\sqrt{\frac{1}{xz}}\)

Đặt \(a=\frac{1}{x};b=\frac{1}{y};c=\frac{1}{z}\)

Khi đó \(a+b+c=1\)

BĐT <=>\(\sqrt{a+bc}+\sqrt{b+ac}+\sqrt{c+ab}\ge1+\sqrt{ab}+\sqrt{bc}+\sqrt{ac}\)

Ta có \(\sqrt{a+bc}=\sqrt{a\left(a+b+c\right)+bc}=\sqrt{\left(a+b\right)\left(a+c\right)}\ge\sqrt{\left(a+\sqrt{bc}\right)^2}=a+\sqrt{bc}\)

Khi đó \(VT\ge a+b+c+\sqrt{ab}+\sqrt{bc}+\sqrt{ac}=1+\sqrt{ab}+\sqrt{bc}+\sqrt{ac}=VP\)(ĐPCM)

Dấu bằng xảy ra khi x=y=z=3

1 tháng 3 2020

BĐT cho tương đương với 

\(\sqrt{a+bc}+\sqrt{b+ca}+\sqrt{c+ab}\ge1+\sqrt{ab}+\sqrt{bc}+\sqrt{ca}\)

Với \(a=\frac{1}{x};b=\frac{1}{y};c=\frac{1}{z};a+b+c=1\)

Ta có:

\(\sqrt{a+bc}=\sqrt{a\left(a+b+c\right)+bc}\)

\(=\sqrt{a^2+a\left(b+c\right)+bc}\ge\sqrt{a^2+2a\sqrt{bc}+bc}=a+\sqrt{bc}\)

Tương tự

\(\sqrt{b+ca}\ge b+\sqrt{ca};\sqrt{c+ab}\ge c+\sqrt{ab}\)

Từ đó ta có đpcm

Dấu "=" xảy ra khi x=y=z=3

10 tháng 9 2016

Ta có : \(\frac{1}{x}+\frac{1}{y}+\frac{1}{z}=1\Leftrightarrow xy+yz+zx=xyz\)

\(\sqrt{x+yz}+\sqrt{y+zx}+\sqrt{z+xy}\ge\sqrt{xyz}+\sqrt{x}+\sqrt{y}+\sqrt{z}\)

Bình phương vế trái : 

\(\left(\sqrt{x+yz}+\sqrt{y+zx}+\sqrt{z+xy}\right)^2\)

\(=\left(x+y+z+xy+yz+zx\right)+2\left(\sqrt{x+yz}.\sqrt{y+zx}+\sqrt{y+zx}.\sqrt{z+xy}+\sqrt{z+xy}.\sqrt{x+yz}\right)\)Bình phương vế phải : 

\(\left(\sqrt{xyz}+\sqrt{x}+\sqrt{y}+\sqrt{z}\right)^2=\left(xyz+x+y+z\right)+2\left(x\sqrt{yz}+y\sqrt{xz}+z\sqrt{xy}+\sqrt{xy}+\sqrt{yz}+\sqrt{zx}\right)\)

Suy ra cần phải chứng minh : \(\sqrt{x+yz}.\sqrt{y+zx}+\sqrt{y+zx}.\sqrt{z+xy}+\sqrt{z+xy}.\sqrt{x+yz}\ge x\sqrt{yz}+y\sqrt{xz}+z\sqrt{xy}+\sqrt{x}+\sqrt{y}+\sqrt{z}\)(*)

Thật vậy, theo bđt Bunhiacopxki ta có : \(\sqrt{x+yz}.\sqrt{y+zx}\ge\sqrt{xy}+z\sqrt{xy}\)

\(\sqrt{y+zx}.\sqrt{z+xy}\ge\sqrt{yz}+x\sqrt{yz}\)

\(\sqrt{z+xy}.\sqrt{x+yz}\ge\sqrt{xz}+y\sqrt{xz}\)

Cộng các bđt trên theo vế ta chứng minh được (*) đúng.

Vậy bđt ban đầu được chứng minh.

 

 

14 tháng 2 2017

Ý tưởng khác

Cũng từ giả thiết suy ra \(xyz=xy+yz+xz\)

Suy ra \(\sqrt{x+yz}=\sqrt{\frac{x^2+xyz}{x}}=\sqrt{\frac{x^2+xy+yz+xz}{x}}=\sqrt{\frac{\left(x+y\right)\left(x+z\right)}{x}}\)

Theo BĐT Cauchy-Schwarz ta có \(\sqrt{\left(x+y\right)\left(x+z\right)}\ge x+\sqrt{yz}\) do đó:

\(\sqrt{x+yz}=\sqrt{\frac{\left(x+y\right)\left(x+z\right)}{x}}\ge\frac{x+\sqrt{yz}}{x}=\sqrt{x}+\sqrt{\frac{yz}{x}}\)

Tương tự cho 2 BĐT còn lại \(\sqrt{y+xz}\ge\sqrt{y}+\sqrt{\frac{xz}{y}};\sqrt{z+xy}\ge\sqrt{z}+\sqrt{\frac{xy}{z}}\)

Cộng theo vế 3 BĐT được \(VT\ge\sqrt{x}+\sqrt{\frac{yz}{x}}+\sqrt{y}+\sqrt{\frac{xz}{y}}+\sqrt{z}+\sqrt{\frac{xy}{z}}\)

\(\Leftrightarrow VT\ge\sqrt{x}+\sqrt{y}+\sqrt{z}+\frac{xy+yz+xz}{\sqrt{xyz}}\)

\(\Leftrightarrow VT\ge\sqrt{x}+\sqrt{y}+\sqrt{z}+\sqrt{xyz}\) (Đpcm)

16 tháng 1 2020

\(VT\ge\frac{9}{\Sigma_{cyc}\sqrt{xy+x+y}}\ge\frac{9}{\sqrt{\left(1+1+1\right)\left(2x+2y+2z+xy+yz+zx\right)}}\ge\frac{9}{\sqrt{3\left[6+\frac{\left(x+y+z\right)^2}{3}\right]}}=\sqrt{3}\)

19 tháng 8 2015

Bạn thiếu giả thiết \(x,y,z>0\) nhé.

Theo giả thiết \(xyz=xy+yz+zx.\)  Từ đó ta có\(\sqrt{x+yz}=\sqrt{\frac{x^2+xyz}{x}}=\sqrt{\frac{x^2+xy+yz+zx}{x}}=\sqrt{\frac{\left(x+y\right)\left(x+z\right)}{x}}.\)

Theo bất đẳng thức Bunhiacốpxki, \(\sqrt{\left(x+y\right)\left(x+z\right)}\ge x+\sqrt{yz}.\)  Do đó

\(\sqrt{x+yz}=\sqrt{\frac{\left(x+y\right)\left(x+z\right)}{x}}\ge\frac{x+\sqrt{yz}}{\sqrt{x}}=\sqrt{x}+\sqrt{\frac{yz}{x}}\), hay ta có \(\sqrt{x+yz}\ge\sqrt{x}+\sqrt{\frac{yz}{x}}.\) 

Tương tự ta có hai bất đẳng thức nữa\(\sqrt{y+zx}\ge\sqrt{y}+\sqrt{\frac{xz}{y}},\sqrt{z+xy}\ge\sqrt{z}+\sqrt{\frac{xy}{z}}\).  Cộng cả ba bất đẳng thức lại cho ta

\(\sqrt{x+yz}+\sqrt{y+zx}+\sqrt{z+xy}\ge\sqrt{x}+\sqrt{\frac{yz}{x}}+\sqrt{y}+\sqrt{\frac{zx}{y}}+\sqrt{z}+\sqrt{\frac{xy}{z}}\)

\(\Leftrightarrow\sqrt{x+yz}+\sqrt{y+zx}+\sqrt{z+xy}\ge\sqrt{x}+\sqrt{y}+\sqrt{z}+\left(\sqrt{\frac{yz}{x}}+\sqrt{\frac{zx}{y}}+\sqrt{\frac{xy}{z}}\right)\)

\(\Leftrightarrow\sqrt{x+yz}+\sqrt{y+zx}+\sqrt{z+xy}\ge\sqrt{x}+\sqrt{y}+\sqrt{z}+\frac{xy+yz+zx}{\sqrt{xyz}}\)

\(\Leftrightarrow\sqrt{x+yz}+\sqrt{y+zx}+\sqrt{z+xy}\ge\sqrt{x}+\sqrt{y}+\sqrt{z}+\sqrt{xyz}.\)    (ĐPCM)

 

18 tháng 8 2015

hừm,, dài quá nên hơi " ngán" hi hi

2 tháng 4 2021

Đặt \(A=\sqrt{x+yz}+\sqrt{y+zx}+\sqrt{z+xy}\)

Ta có:

\(x^2+xy+yz+zx=x+xyz=x\left(x+yz\right)\)

\(\Rightarrow\frac{x\left(x+yz\right)}{x}=\frac{x^2+xy+yz+zx}{x}\)

\(\Leftrightarrow x+yz=\frac{x^2+xy+yz+zx}{x}=\frac{\left(x^2+xy\right)+\left(yz+zx\right)}{x}=\frac{\left(x+z\right)\left(x+y\right)}{x}\)

\(\Rightarrow\sqrt{x+yz}=\sqrt{\frac{\left(x+y\right)\left(x+z\right)}{x}}\)

Vì x, y, z >0 nên áp dụng bất đẳng thức Bunhiacopxki cho 2 số dương, ta được:

\(\left(x+y\right)\left(x+z\right)\ge\left(\sqrt{x^2}.+\sqrt{yz}\right)^2\)

\(\Rightarrow\sqrt{\left(x+y\right)\left(x+z\right)}\ge x+\sqrt{yz}\)

\(\Rightarrow\sqrt{\frac{\left(x+y\right)\left(x+z\right)}{x}}\ge\frac{x+\sqrt{yz}}{\sqrt{x}}\)

Do đó \(\sqrt{x+yz}\ge\frac{x+\sqrt{yz}}{\sqrt{x}}\left(1\right)\)

Chứng minh tương tự, ta được:

\(\sqrt{y+xz}\ge\frac{y+\sqrt{xz}}{\sqrt{y}}\left(2\right)\)

Chứng minh tương tự, ta được:

\(\sqrt{z+xy}\ge\frac{z+\sqrt{xy}}{\sqrt{z}}\left(3\right)\)

Từ (1), (2) và (3), ta được:

\(\sqrt{x+yz}+\sqrt{y+zx}+\sqrt{z+xy}\)\(\ge\frac{x+\sqrt{yz}}{\sqrt{x}}+\frac{y+\sqrt{zx}}{\sqrt{y}}+\frac{z+\sqrt{xy}}{\sqrt{z}}\)

\(\Leftrightarrow A\ge\sqrt{x}+\sqrt{\frac{yz}{x}}+\sqrt{y}+\sqrt{\frac{xz}{y}}+\sqrt{z}+\sqrt{\frac{xy}{z}}\)

\(\Leftrightarrow A\ge\sqrt{x}+\sqrt{y}+\sqrt{z}+\frac{yz+zx+xy}{\sqrt{xyz}}\)

 \(\Leftrightarrow A\ge\sqrt{x}+\sqrt{y}+\sqrt{z}+\frac{xyz}{\sqrt{xyz}}\)(vì \(xy+yz+zx=xyz\))

\(\Leftrightarrow A\ge\sqrt{x}+\sqrt{y}+\sqrt{z}+\sqrt{xyz}\)(điều phải chứng minh).

Dấu bằng xảy ra.

\(\Leftrightarrow\hept{\begin{cases}x=y=z>0\\xy+yz+zx=xyz\end{cases}}\Leftrightarrow x=y=z=3\)

Vậy với x, y, z là các số thực dương thỏa mãn xy + yz + zx =xyz thì:

\(\sqrt{x+yz}+\sqrt{y+zx}+\sqrt{z+xy}\ge\sqrt{x}+\sqrt{y}+\sqrt{z}+\sqrt{xyz}\).

\(\)

3 tháng 4 2020

helloo

3 tháng 4 2020

Ta có \(1+x^2=x^2+xy+yz+xz=\left(x+z\right)\left(x+y\right)\)

Khi đó BĐT <=>

 \(\frac{1}{\left(x+y\right)\left(x+z\right)}+\frac{1}{\left(y+z\right)\left(x+z\right)}+\frac{1}{\left(x+y\right)\left(y+z\right)}\ge\frac{2}{3}\left(\frac{x}{\sqrt{\left(x+z\right)\left(x+y\right)}}+...\right)\)

<=> \(\frac{x+y+z}{\left(x+y\right)\left(y+z\right)\left(x+z\right)}\ge\frac{1}{3}\left(\frac{x\sqrt{y+z}+y\sqrt{x+z}+z\sqrt{x+y}}{\sqrt{\left(x+y\right)\left(y+z\right)\left(x+z\right)}}\right)^3\)

<=>\(\left(x+y+z\right)\sqrt{\left(x+y\right)\left(x+z\right)\left(y+z\right)}\ge\frac{1}{3}\left(x\sqrt{y+z}+y\sqrt{x+z}+z\sqrt{x+y}\right)^3\)

<=> \(\left(x+y+z\right)\sqrt{\left(x+y\right)\left(y+z\right)\left(x+z\right)}\ge\frac{1}{3}\left(\sqrt{x\left(1-yz\right)}+\sqrt{y\left(1-xz\right)}+\sqrt{z\left(1-xy\right)}\right)^3\)(1)

Xét \(\left(x+y\right)\left(y+z\right)\left(x+z\right)\ge\frac{8}{9}\left(x+y+z\right)\left(xy+yz+xz\right)\)

<=> \(9\left[xy\left(x+y\right)+yz\left(y+z\right)+xz\left(x+z\right)+2xyz\right]\ge8\left(xy\left(x+y\right)+xz\left(x+z\right)+yz\left(y+z\right)+3xyz\right)\)

<=> \(xy\left(y+x\right)+yz\left(y+z\right)+xz\left(x+z\right)\ge6xyz\)

<=> \(x\left(y-z\right)^2+z\left(x-y\right)^2+y\left(x-z\right)^2\ge0\)luôn đúng

Khi đó (1) <=> 

\(\left(x+y+z\right).\frac{2\sqrt{2}}{3}.\sqrt{x+y+z}\ge\frac{1}{3}\left(\sqrt{x\left(1-yz\right)}+....\right)^3\) 

<=> \(\sqrt{2\left(x+y+z\right)}\ge\sqrt{x\left(1-yz\right)}+\sqrt{y\left(1-xz\right)}+\sqrt{z\left(1-xy\right)}\)

Áp dụng buniacopxki cho vế phải ta có 

\(\sqrt{x\left(1-yz\right)}+\sqrt{y\left(1-xz\right)}+\sqrt{z\left(1-xy\right)}\le\sqrt{\left(x+y+z\right)\left(3-xy-yz-xz\right)}\)

                                                                                                       \(=\sqrt{2\left(x+y+z\right)}\)

=> BĐT được CM

Dấu bằng xảy ra khi \(x=y=z=\frac{1}{\sqrt{3}}\)

22 tháng 6 2016

Chứng minh cái j vậy bạn

22 tháng 6 2016

cmr \(\frac{\sqrt{1+x^3+y^3}}{xy}+\frac{\sqrt{1+y^3+z^3}}{yz}+\frac{\sqrt{1+z^3+x^3}}{zx}\)

15 tháng 9 2023

Ta cần chứng minh:\(\dfrac{1}{\sqrt{x+y+xy}}+\dfrac{1}{\sqrt{y+z+yz}}+\dfrac{1}{\sqrt{z+x+zx}}\ge\sqrt{3}\)

Áp dụng bất đẳng thức Bunhiacopxki, ta được:

 

\(\dfrac{1}{\sqrt{x+y+xy}}+\dfrac{1}{\sqrt{y+z+yz}}+\dfrac{1}{\sqrt{z+x+zx}}\ge\dfrac{9}{\sqrt{x+y+xy}+\sqrt{y+z+yz}+\sqrt{z+x+zx}}\)

Mặt khác, ta có:

\(\left(\sqrt{x+y+xy}+\sqrt{y+z+yz}+\sqrt{z+x+zx}\right)^2\le3\left(\left(x+y+xy\right)+\left(y+z+yz\right)+\left(z+x+zx\right)\right)\)

\(\Leftrightarrow\left(\sqrt{x+y+xy}+\sqrt{y+z+yz}+\sqrt{z+x+zx}\right)^2\le3\left(6+xy+yz+zx\right)\)Lại có:

\(xy+yz+zx\le\dfrac{\left(x+y+z\right)^2}{3}=\dfrac{9}{3}=3\)

\(\Rightarrow\left(\sqrt{x+y+xy}+\sqrt{y+z+yz}+\sqrt{z+x+zx}\right)^2\le3\left(6+3\right)=27\)

15 tháng 9 2023

\(\Rightarrow\sqrt{x+y+xy}+\sqrt{y+z+yz}+\sqrt{z+x+zx}\le3\sqrt{3}\)

\(\Rightarrow\dfrac{9}{\sqrt{x+y+xy}+\sqrt{y+z+yz}+\sqrt{z+x+zx}}\ge\dfrac{9}{3\sqrt{3}}=\sqrt{3}\)

Do đó \(\dfrac{1}{\sqrt{x+y+xy}}+\dfrac{1}{\sqrt{y+z+yz}}+\dfrac{1}{\sqrt{z+x+zx}}\ge\sqrt{3}\)

Dấu bằng xảy ra \(\Leftrightarrow x=y=z=1\).

NV
2 tháng 4 2020

Lời giải bài này khá dài, làm biếng gõ

Bạn lên google search "đề thi vào 10 chuyên khtn" nhé, đây là bài BĐT trong đề vòng 1 chuyên KHTN năm 2019

2 tháng 4 2020

Ta có:

\( 1 + {x^2} = \left( {x + y} \right)\left( {x + z} \right)\\ 1 + {y^2} = \left( {x + y} \right)\left( {y + z} \right)\\ 1 + {z^2} = \left( {x + z} \right)\left( {y + z} \right) \)

Nên: \(\dfrac{1}{{1 + {x^2}}} + \dfrac{1}{{1 + {y^2}}} + \dfrac{1}{{1 + {z^2}}} = \dfrac{1}{{\left( {x + y} \right)\left( {x + z} \right)}} + \dfrac{1}{{\left( {x + y} \right)\left( {y + z} \right)}} + \dfrac{1}{{\left( {x + z} \right)\left( {y + z} \right)}} = \dfrac{{2\left( {x + y + z} \right)}}{{\left( {x + y} \right)\left( {y + z} \right)\left( {x + z} \right)}}\)

\( \dfrac{x}{{\sqrt {1 + {x^2}} }} + \dfrac{y}{{\sqrt {1 + {y^2}} }} + \dfrac{z}{{\sqrt {1 + {z^2}} }} = \dfrac{x}{{\sqrt {\left( {x + y} \right)\left( {x + z} \right)} }} + \dfrac{y}{{\sqrt {\left( {x + y} \right)\left( {y + z} \right)} }} + \dfrac{z}{{\left( {x + z} \right)\left( {y + z} \right)}}\\ \dfrac{x}{{\sqrt {1 + {x^2}} }} + \dfrac{y}{{\sqrt {1 + {y^2}} }} + \dfrac{z}{{\sqrt {1 + {z^2}} }} \le \dfrac{1}{2}\left( {\dfrac{x}{{x + y}} + \dfrac{x}{{x + z}} + \dfrac{y}{{x + y}} + \dfrac{y}{{y + z}} + \dfrac{z}{{x + z}} + \dfrac{z}{{y + z}}} \right) \)

Mặt khác, áp dụng $Bunhia$ ta có:

\({\left( {\dfrac{x}{{\sqrt {1 + {x^2}} }} + \dfrac{y}{{\sqrt {1 + {y^2}} }} + \dfrac{z}{{\sqrt {1 + {z^2}} }}} \right)^2} \le \left( {x + y + z} \right)\left( {\dfrac{x}{{1 + {x^2}}} + \dfrac{y}{{1 + {y^2}}} + \dfrac{z}{{1 + {z^2}}}} \right) = M\)

Với \(M = \dfrac{{2\left( {x + y + z} \right)\left( {xy + yz + xz} \right)}}{{\left( {x + y} \right)\left( {x + z} \right)\left( {y + z} \right)}} = \dfrac{{2\left( {x + y + z} \right)}}{{\left( {x + y} \right)\left( {x + z} \right)\left( {y + z} \right)}}\)

Lại có:

\( VP = \dfrac{2}{3}{\left( {\dfrac{x}{{1 + {x^2}}} + \dfrac{y}{{1 + {y^2}}} + \dfrac{z}{{1 + {z^2}}}} \right)^3} = \dfrac{2}{3}{\left( {\dfrac{1}{{1 + {x^2}}} + \dfrac{1}{{1 + {y^2}}} + \dfrac{1}{{1 + {z^2}}}} \right)^2}\\ VP \le \dfrac{{4\left( {x + y + z} \right)}}{{3\left( {x + y} \right)\left( {x + z} \right)\left( {y + z} \right)}}.\dfrac{3}{2} = \dfrac{{2\left( {x + y + z} \right)}}{{\left( {x + y} \right)\left( {x + z} \right)\left( {y + z} \right)}} = \dfrac{1}{{1 + {x^2}}} + \dfrac{1}{{1 + {y^2}}} + \dfrac{1}{{1 + {z^2}}} \)

Vậy \(\dfrac{1}{{1 + {x^2}}} + \dfrac{1}{{1 + {y^2}}} + \dfrac{1}{{1 + {z^2}}} \ge \dfrac{3}{2}{\left( {\dfrac{x}{{\sqrt {1 + {x^2}} }} + \dfrac{y}{{\sqrt {1 + {y^2}} }} + \dfrac{z}{{\sqrt {1 + {z^2}} }}} \right)^2}\)

Dấu \("= "\) xảy ra khi \(x=y=z=\dfrac{1}{\sqrt{3}}\)